Finding the inverse laplace transform of (2/(s+2)^4) using Convolution theorem.

Click For Summary
The discussion revolves around finding the inverse Laplace transform of (2/(s+2)^4) using the convolution theorem. Participants express concerns about the accuracy of the provided table of identities, suggesting that it may contain flaws. There is a suggestion to derive the identity for the inverse transform instead of relying on the potentially incorrect table. The integral involved in the convolution approach is acknowledged as messy, but some participants argue it is manageable. Overall, the conversation emphasizes the importance of using accurate identities and exploring alternative methods for solving the problem.
theBEAST
Messages
361
Reaction score
0

Homework Statement


Find the inverse laplace transform of (2/(s+2)^4) using the given table of identities:

Homework Equations


Here are the given identities:
FWlY5.png


The Attempt at a Solution


Alright, I realize that there is a simple identity that I can use with a factorial symbol, but this identity is not on our formula sheet, so I decided to try it with convolution theorem.

xiuk2.jpg


What did I do wrong and also is there an easier way given the table of identities? Thanks!
 
Physics news on Phys.org
Where did you get that flawed table? Discard it.

G(s)H(s) \rightarrow \int_0^t g(t-\tau)h(\tau) \mathop{d\tau}

Clearly both functions should depend on the integration variable.
 
Last edited:
lurflurf said:
Where did you get that flawed table? Discard it.

G(s)H(s) \rightarrow \int_0^t g(t-\tau)h(\tau) d\tau

Clearly both functions should depend on the integration variable.

Ahh, thanks!

This is the table that we get for our midterms and final lol

Is there an easier way to solve this with another identity? The integral is pretty messy.
 
That integral is not so bad, and you wanted to avoid using

\frac{1}{(s+a)^{n+1}} \rightarrow \frac{t^n}{n!} e^{-a \mathop{t}}

you could derive it. It helps to have a longer table with fewer errors.
 
Last edited:
Question: A clock's minute hand has length 4 and its hour hand has length 3. What is the distance between the tips at the moment when it is increasing most rapidly?(Putnam Exam Question) Answer: Making assumption that both the hands moves at constant angular velocities, the answer is ## \sqrt{7} .## But don't you think this assumption is somewhat doubtful and wrong?

Similar threads

  • · Replies 10 ·
Replies
10
Views
2K
  • · Replies 1 ·
Replies
1
Views
1K
  • · Replies 1 ·
Replies
1
Views
2K
  • · Replies 4 ·
Replies
4
Views
3K
Replies
2
Views
2K
  • · Replies 8 ·
Replies
8
Views
2K
  • · Replies 2 ·
Replies
2
Views
3K
Replies
5
Views
2K
  • · Replies 4 ·
Replies
4
Views
3K
  • · Replies 1 ·
Replies
1
Views
2K